Can ratio of smooth numbers approach 1?











up vote
4
down vote

favorite












Can the ratio of $q$-smooth numbers ever approach one? Asked more rigorously, can we ever have
$$1in overline{{a/b; :; a,b ;text{ are } qtext{-smooth}}}=overline{{prod_{p_ile q}p_i^{alpha_i}; :; (a_1, dots, a_{pi(q)})inmathbb{Z}^{pi(q)}}};?$$
If $q=2$, the answer is obviously no because
$$1notin overline{{2^{alpha}; :; alphainmathbb{Z}}}={0}cup {2^{alpha}; :; alphainmathbb{Z}}$$
however beyond this the problem is not easy. The reason I would like to know this is because this would imply that
$$G(q)=inf {a/b; :; a>b,; a,b ; qtext{-smooth}}>1.$$
I suspect that we may be able to say even further that
$$overline{{prod_{p_ile q}p_i^{alpha_i}; :; (a_1, dots, a_{pi(q)})inmathbb{Z}^{pi(q)}}}={0}cup {prod_{p_ile q}p_i^{alpha_i}; :; (a_1, dots, a_{pi(q)})inmathbb{Z}^{pi(q)}}$$
which would clearly imply the result.










share|cite|improve this question






















  • Quite amusingly, this is on the sitewide hot questions list.
    – Matt Samuel
    Nov 3 at 22:40










  • @MattSamuel: Probably because people like to hear "smooth" talk? =P
    – user21820
    Nov 4 at 3:02















up vote
4
down vote

favorite












Can the ratio of $q$-smooth numbers ever approach one? Asked more rigorously, can we ever have
$$1in overline{{a/b; :; a,b ;text{ are } qtext{-smooth}}}=overline{{prod_{p_ile q}p_i^{alpha_i}; :; (a_1, dots, a_{pi(q)})inmathbb{Z}^{pi(q)}}};?$$
If $q=2$, the answer is obviously no because
$$1notin overline{{2^{alpha}; :; alphainmathbb{Z}}}={0}cup {2^{alpha}; :; alphainmathbb{Z}}$$
however beyond this the problem is not easy. The reason I would like to know this is because this would imply that
$$G(q)=inf {a/b; :; a>b,; a,b ; qtext{-smooth}}>1.$$
I suspect that we may be able to say even further that
$$overline{{prod_{p_ile q}p_i^{alpha_i}; :; (a_1, dots, a_{pi(q)})inmathbb{Z}^{pi(q)}}}={0}cup {prod_{p_ile q}p_i^{alpha_i}; :; (a_1, dots, a_{pi(q)})inmathbb{Z}^{pi(q)}}$$
which would clearly imply the result.










share|cite|improve this question






















  • Quite amusingly, this is on the sitewide hot questions list.
    – Matt Samuel
    Nov 3 at 22:40










  • @MattSamuel: Probably because people like to hear "smooth" talk? =P
    – user21820
    Nov 4 at 3:02













up vote
4
down vote

favorite









up vote
4
down vote

favorite











Can the ratio of $q$-smooth numbers ever approach one? Asked more rigorously, can we ever have
$$1in overline{{a/b; :; a,b ;text{ are } qtext{-smooth}}}=overline{{prod_{p_ile q}p_i^{alpha_i}; :; (a_1, dots, a_{pi(q)})inmathbb{Z}^{pi(q)}}};?$$
If $q=2$, the answer is obviously no because
$$1notin overline{{2^{alpha}; :; alphainmathbb{Z}}}={0}cup {2^{alpha}; :; alphainmathbb{Z}}$$
however beyond this the problem is not easy. The reason I would like to know this is because this would imply that
$$G(q)=inf {a/b; :; a>b,; a,b ; qtext{-smooth}}>1.$$
I suspect that we may be able to say even further that
$$overline{{prod_{p_ile q}p_i^{alpha_i}; :; (a_1, dots, a_{pi(q)})inmathbb{Z}^{pi(q)}}}={0}cup {prod_{p_ile q}p_i^{alpha_i}; :; (a_1, dots, a_{pi(q)})inmathbb{Z}^{pi(q)}}$$
which would clearly imply the result.










share|cite|improve this question













Can the ratio of $q$-smooth numbers ever approach one? Asked more rigorously, can we ever have
$$1in overline{{a/b; :; a,b ;text{ are } qtext{-smooth}}}=overline{{prod_{p_ile q}p_i^{alpha_i}; :; (a_1, dots, a_{pi(q)})inmathbb{Z}^{pi(q)}}};?$$
If $q=2$, the answer is obviously no because
$$1notin overline{{2^{alpha}; :; alphainmathbb{Z}}}={0}cup {2^{alpha}; :; alphainmathbb{Z}}$$
however beyond this the problem is not easy. The reason I would like to know this is because this would imply that
$$G(q)=inf {a/b; :; a>b,; a,b ; qtext{-smooth}}>1.$$
I suspect that we may be able to say even further that
$$overline{{prod_{p_ile q}p_i^{alpha_i}; :; (a_1, dots, a_{pi(q)})inmathbb{Z}^{pi(q)}}}={0}cup {prod_{p_ile q}p_i^{alpha_i}; :; (a_1, dots, a_{pi(q)})inmathbb{Z}^{pi(q)}}$$
which would clearly imply the result.







number-theory limits elementary-number-theory






share|cite|improve this question













share|cite|improve this question











share|cite|improve this question




share|cite|improve this question










asked Nov 3 at 21:19









Will Fisher

3,487629




3,487629












  • Quite amusingly, this is on the sitewide hot questions list.
    – Matt Samuel
    Nov 3 at 22:40










  • @MattSamuel: Probably because people like to hear "smooth" talk? =P
    – user21820
    Nov 4 at 3:02


















  • Quite amusingly, this is on the sitewide hot questions list.
    – Matt Samuel
    Nov 3 at 22:40










  • @MattSamuel: Probably because people like to hear "smooth" talk? =P
    – user21820
    Nov 4 at 3:02
















Quite amusingly, this is on the sitewide hot questions list.
– Matt Samuel
Nov 3 at 22:40




Quite amusingly, this is on the sitewide hot questions list.
– Matt Samuel
Nov 3 at 22:40












@MattSamuel: Probably because people like to hear "smooth" talk? =P
– user21820
Nov 4 at 3:02




@MattSamuel: Probably because people like to hear "smooth" talk? =P
– user21820
Nov 4 at 3:02










2 Answers
2






active

oldest

votes

















up vote
6
down vote



accepted










Yes, if $qge 3$.



This is because $log_2(3)$ is irrational, so the fractional parts of $log_2(3^n)$ lie densely in $[0,1)$. In particular they can become arbitrarily small.






share|cite|improve this answer






























    up vote
    6
    down vote













    Yes. We can just consider fractions of the form $frac {3^a}{2^b}$. The base $2$ log of this is $alog_2(3)-b$. Because $log_2(3)$ is irrational, we can choose $a,b$ to make this difference arbitrarily close to $0$, which means the ratio can be arbitrarily close to $1$.






    share|cite|improve this answer





















      Your Answer





      StackExchange.ifUsing("editor", function () {
      return StackExchange.using("mathjaxEditing", function () {
      StackExchange.MarkdownEditor.creationCallbacks.add(function (editor, postfix) {
      StackExchange.mathjaxEditing.prepareWmdForMathJax(editor, postfix, [["$", "$"], ["\\(","\\)"]]);
      });
      });
      }, "mathjax-editing");

      StackExchange.ready(function() {
      var channelOptions = {
      tags: "".split(" "),
      id: "69"
      };
      initTagRenderer("".split(" "), "".split(" "), channelOptions);

      StackExchange.using("externalEditor", function() {
      // Have to fire editor after snippets, if snippets enabled
      if (StackExchange.settings.snippets.snippetsEnabled) {
      StackExchange.using("snippets", function() {
      createEditor();
      });
      }
      else {
      createEditor();
      }
      });

      function createEditor() {
      StackExchange.prepareEditor({
      heartbeatType: 'answer',
      convertImagesToLinks: true,
      noModals: true,
      showLowRepImageUploadWarning: true,
      reputationToPostImages: 10,
      bindNavPrevention: true,
      postfix: "",
      imageUploader: {
      brandingHtml: "Powered by u003ca class="icon-imgur-white" href="https://imgur.com/"u003eu003c/au003e",
      contentPolicyHtml: "User contributions licensed under u003ca href="https://creativecommons.org/licenses/by-sa/3.0/"u003ecc by-sa 3.0 with attribution requiredu003c/au003e u003ca href="https://stackoverflow.com/legal/content-policy"u003e(content policy)u003c/au003e",
      allowUrls: true
      },
      noCode: true, onDemand: true,
      discardSelector: ".discard-answer"
      ,immediatelyShowMarkdownHelp:true
      });


      }
      });














       

      draft saved


      draft discarded


















      StackExchange.ready(
      function () {
      StackExchange.openid.initPostLogin('.new-post-login', 'https%3a%2f%2fmath.stackexchange.com%2fquestions%2f2983417%2fcan-ratio-of-smooth-numbers-approach-1%23new-answer', 'question_page');
      }
      );

      Post as a guest
































      2 Answers
      2






      active

      oldest

      votes








      2 Answers
      2






      active

      oldest

      votes









      active

      oldest

      votes






      active

      oldest

      votes








      up vote
      6
      down vote



      accepted










      Yes, if $qge 3$.



      This is because $log_2(3)$ is irrational, so the fractional parts of $log_2(3^n)$ lie densely in $[0,1)$. In particular they can become arbitrarily small.






      share|cite|improve this answer



























        up vote
        6
        down vote



        accepted










        Yes, if $qge 3$.



        This is because $log_2(3)$ is irrational, so the fractional parts of $log_2(3^n)$ lie densely in $[0,1)$. In particular they can become arbitrarily small.






        share|cite|improve this answer

























          up vote
          6
          down vote



          accepted







          up vote
          6
          down vote



          accepted






          Yes, if $qge 3$.



          This is because $log_2(3)$ is irrational, so the fractional parts of $log_2(3^n)$ lie densely in $[0,1)$. In particular they can become arbitrarily small.






          share|cite|improve this answer














          Yes, if $qge 3$.



          This is because $log_2(3)$ is irrational, so the fractional parts of $log_2(3^n)$ lie densely in $[0,1)$. In particular they can become arbitrarily small.







          share|cite|improve this answer














          share|cite|improve this answer



          share|cite|improve this answer








          edited Nov 3 at 21:44

























          answered Nov 3 at 21:26









          Henning Makholm

          234k16299532




          234k16299532






















              up vote
              6
              down vote













              Yes. We can just consider fractions of the form $frac {3^a}{2^b}$. The base $2$ log of this is $alog_2(3)-b$. Because $log_2(3)$ is irrational, we can choose $a,b$ to make this difference arbitrarily close to $0$, which means the ratio can be arbitrarily close to $1$.






              share|cite|improve this answer

























                up vote
                6
                down vote













                Yes. We can just consider fractions of the form $frac {3^a}{2^b}$. The base $2$ log of this is $alog_2(3)-b$. Because $log_2(3)$ is irrational, we can choose $a,b$ to make this difference arbitrarily close to $0$, which means the ratio can be arbitrarily close to $1$.






                share|cite|improve this answer























                  up vote
                  6
                  down vote










                  up vote
                  6
                  down vote









                  Yes. We can just consider fractions of the form $frac {3^a}{2^b}$. The base $2$ log of this is $alog_2(3)-b$. Because $log_2(3)$ is irrational, we can choose $a,b$ to make this difference arbitrarily close to $0$, which means the ratio can be arbitrarily close to $1$.






                  share|cite|improve this answer












                  Yes. We can just consider fractions of the form $frac {3^a}{2^b}$. The base $2$ log of this is $alog_2(3)-b$. Because $log_2(3)$ is irrational, we can choose $a,b$ to make this difference arbitrarily close to $0$, which means the ratio can be arbitrarily close to $1$.







                  share|cite|improve this answer












                  share|cite|improve this answer



                  share|cite|improve this answer










                  answered Nov 3 at 21:27









                  Ross Millikan

                  285k23195363




                  285k23195363






























                       

                      draft saved


                      draft discarded



















































                       


                      draft saved


                      draft discarded














                      StackExchange.ready(
                      function () {
                      StackExchange.openid.initPostLogin('.new-post-login', 'https%3a%2f%2fmath.stackexchange.com%2fquestions%2f2983417%2fcan-ratio-of-smooth-numbers-approach-1%23new-answer', 'question_page');
                      }
                      );

                      Post as a guest




















































































                      這個網誌中的熱門文章

                      Tangent Lines Diagram Along Smooth Curve

                      Yusuf al-Mu'taman ibn Hud

                      Zucchini